2016 AMC 10B 考试题目

Scroll down and press Start to try the exam! Or, go to the printable PDF, answer key, or solutions.

Used with permission of the Mathematical Association of America.

考试时间还剩下:

1:15:00

1.

What is the value of 2a1+a12a\dfrac{2a^{-1}+\frac{a^{-1}}{2}}{a} when a=12?a= \tfrac{1}{2}?

 1 \ 1

 2 \ 2

 52 \ \dfrac{5}{2}

 10 \ 10

 20 \ 20

Answer: D
Solution(s):

The expression is equivalent to 2a2+a22=2.5(a1)2.2a^{-2}+\frac{a^{-2}}{2} = 2.5 (a^{-1})^2. Then a1a^{-1} is equal to 112=2,\dfrac{1}{\frac 12} = 2, so our expression is equal to 2.522=10.2.5\cdot 2^2 = 10.

Thus, the correct answer is D.

2.

If nm=n3m2,n\heartsuit m=n^3m^2, what is 2442?\frac{2\heartsuit 4}{4\heartsuit 2}?

 14 \ \dfrac{1}{4}

 12 \ \dfrac{1}{2}

 1 \ 1

 2 \ 2

 4 \ 4

Answer: B
Solution(s):

For any nonzero a,b,a,b, we have abba=a3b2b3a2=ab.\dfrac{a\heartsuit b}{b\heartsuit a} = \dfrac{a^3b^2}{b^3 a^2} = \dfrac ab . Using a=2,b=4,a=2,b=4, we get that 24=12.\frac{ 2}{4} = \frac 12.

Thus, the correct answer is B.

3.

Let x=2016.x=-2016. What is the value of xxxx?\Bigg\vert\Big\vert |x|-x\Big\vert-|x|\Bigg\vert-x?

 2016 \ -2016

 0 \ 0

 2016 \ 2016

 4032 \ 4032

 6048 \ 6048

Answer: D
Solution(s):

Observe that: xxxx=xxxx=2xxx\begin{align*} &\Bigg\vert\Big\vert |x|-x\Big\vert-|x|\Bigg\vert-x\\ &=\Bigg\vert \Big\vert -x-x\Big\vert-|x|\Bigg\vert-x \\ &=\Bigg\vert\Big\vert -2x\Big\vert-|x|\Bigg\vert-x \end{align*} since x<0x < 0 implies that x=x.-x = |x|. Substituting values, we can see that 40322016(2016)|4032-2016|-(-2016) =2016+2016= 2016 +2016 =4032.= 4032.

Thus, the correct answer is D.

4.

Zoey read 1515 books, one at a time. The first book took her 11 day to read, the second book took her 22 days to read, the third book took her 33 days to read, and so on, with each book taking her 11 more day to read than the previous book. Zoey finished the first book on a Monday, and the second on a Wednesday. On what day the week did she finish her 1515th book?

Sunday

Monday

Wednesday

Friday

Saturday

Answer: B
Solution(s):

The number of days it takes to read 1515 books is 15162=120.\dfrac{ 15\cdot 16}{2} =120. Therefore, it is 1201=119120-1=119 days after the first book is read. This is a multiple of 7,7, so the 1515th book was finished on the same day as the 11st book. Therefore, it was finished on a Monday.

Thus, the correct answer is B.

5.

The mean age of Amanda's 44 cousins is 8,8, and their median age is 5.5. What is the sum of the ages of Amanda's youngest and oldest cousins?

 13 \ 13

 16 \ 16

 19 \ 19

 22 \ 22

 25 \ 25

Answer: D
Solution(s):

The sum of everyone's age is 44=32.4\cdot 4 = 32. The median being 55 means that the mean of the middle two cousins is 5.5. This makes the sum of the middle two cousins 2+5=10.2+5=10. Therefore, the sum of the oldest and youngest is 3210=22.32-10=22.

Thus, the correct answer is D.

6.

Laura added two three-digit positive integers. All six digits in these numbers are different. Laura's sum is a three-digit number S.S. What is the smallest possible value for the sum of the digits of S?S?

 1 \ 1

 4 \ 4

 5 \ 5

 15 \ 15

 21 \ 21

Answer: B
Solution(s):

As 300<S<1000,300 \lt S \lt 1000, we know that the sum of the digits must be at least 4.4. This can be seen in 157+243=400.157+243=400.

Thus, the correct answer is B.

7.

The ratio of the measures of two acute angles is 5:4,5:4, and the complement of one of these two angles is twice as large as the complement of the other. What is the sum of the degree measures of the two angles?

 75 \ 75

 90 \ 90

 135 \ 135

 150 \ 150

 270 \ 270

Answer: C
Solution(s):

Let the smaller angle be x.x. Then, the larger angle is 54x.\frac 54 x. Note that their sum is 94x.\frac 94 x.

Their complements are then 90x90-x and 9054x.90 - \frac 54x. Since 90x90-x is larger than 9054x,90-\frac 54 x, we know 90x=2(9054x)90x=18052xx=60.\begin{align*}90 -x &= 2\left(90 - \frac 54 x\right) \\90-x&= 180-\frac 52 x\\x&=60.\end{align*}

Therefore, the sum is 9460=135.\dfrac 94 \cdot 60=135.

Thus, the correct answer is C.

8.

What is the tens digit of 201520162017?2015^{2016}-2017?

 0 \ 0

 1 \ 1

 3 \ 3

 5 \ 5

 8 \ 8

Answer: A
Solution(s):

To find the tens digit, we first need to find the number modulo 100.100. First, we can find 20152016mod100.2015^{2016} \mod 100. We can do this with the Chinese Remainder Theorem by first getting the number mod4\mod 4 and then mod25.\mod 25.

The number 201520160mod252015^{2016} \equiv 0 \mod 25 since it is a multiple of 25.25.

Then, observe that 20152016(1)20162015^{2016} \equiv (-1)^{2016}1mod4. \equiv 1 \mod 4 .

By the Chinese remainder theorem, we can get that our number is congruent to 25mod100.25 \mod 100.

Therefore, 2015201620172015^{2016}-2017 2517mod100\equiv 25 - 17 \mod 1008mod100.\equiv 8 \mod 100 . This means the tens digit is 0.0.

Thus, the correct answer is A.

9.

All three vertices of ABC\bigtriangleup ABC lie on the parabola defined by y=x2,y=x^2, with AA at the origin and BC\overline{BC} parallel to the xx-axis. The area of the triangle is 64.64. What is the length of BC?BC?

 4 \ 4

 6 \ 6

 8 \ 8

 10 \ 10

 16 \ 16

Answer: C
Solution(s):

Let the points be A=(0,0),A=(0,0),B=(x1,y1),B=(x_1,y_1),C=(x2,y2).C=(x_2,y_2).

Then, since BCBC is parallel with the xx-axis, we know y1=y2,y_1=y_2, which we will let be y.y. Then, x12=y=x22,x_1^2 = y=x_2^2, so x12=x22.x_1^2=x_2^2 . This implies that either x1=x2x_1=-x_2 or x1=x2.x_1 = x_2. The second option cannot happen since that would set two points as the same, which would create an area of 0.0. As such, let x=x1=x2.x= -x_1=x_2. Then y=x2y=x^2 also.

Then, the points are A=(0,0),A=(0,0),B=(x,x2),B=(-x,x^2),C=(x,x2).C=(x,x^2). With a base of BC,BC, the length is 2x2x and the height is x2.x^2. This would make the area x3=64.x^3 = 64. Therefore, x=4,x=4, so BC=24=8.BC = 2\cdot 4 = 8.

Thus, the correct answer is C.

10.

A thin piece of wood of uniform density in the shape of an equilateral triangle with side length 33 inches weighs 1212 ounces. A second piece of the same type of wood, with the same thickness, also in the shape of an equilateral triangle, has side length of 55 inches. Which of the following is closest to the weight, in ounces, of the second piece?

 14.0 \ 14.0

 16.0 \ 16.0

 20.0 \ 20.0

 33.3 \ 33.3

 55.6 \ 55.6

Answer: D
Solution(s):

The surface area is increased by a factor of 532\frac 53 ^2 since the side lengths are increased by a factor of 53.\frac 53. Also since the thickness is constant, the volume is scaled up by that much. Then, the wood having the same density makes the weight increase by a factor of 532\frac 53^2 as well, so the new volume is 12259=1003.12 \cdot \dfrac {25}{9} = \dfrac{100}3. This is approximately 33.3.33.3.

Thus, the correct answer is D.

11.

Carl decided to fence in his rectangular garden. He bought 2020 fence posts, placed one on each of the four corners, and spaced out the rest evenly along the edges of the garden, leaving exactly 44 yards between neighboring posts. The longer side of his garden, including the corners, has twice as many posts as the shorter side, including the corners. What is the area, in square yards, of Carl’s garden?

 256 \ 256

 336 \ 336

 384 \ 384

 448 \ 448

 512 \ 512

Answer: B
Solution(s):

Let the number of posts on the longer side be ll and let the number of posts on the shorter side be s.s. Then, l+s+l+s4=20l+s+l+s-4 = 20 since we take out the corners. Thus, 2l+2s=6s=24,2l+2s = 6s = 24, making s=4,l=8.s=4,l=8. Thus, on the shorter side, the corners are 33 posts away and on the longers side, they are 77 posts away. The shorter side is then 34=12,3\cdot 4=12, and the longer side is 74=28.7\cdot 4=28. Thus, the area is 1228=336.12\cdot 28=336.

Thus, the correct answer is B.

12.

Two different numbers are selected at random from {1,2,3,4,5}\{1, 2, 3, 4, 5\} and multiplied together. What is the probability that the product is even?

 0.2 \ 0.2

 0.4 \ 0.4

 0.5 \ 0.5

 0.7 \ 0.7

 0.8 \ 0.8

Answer: D
Solution(s):

The product is odd if an only if both numbers are odd. There are (32)=3\binom 32=3 ways to do this out of a possible (52)=10\binom 52=10 ways. This makes the probability of it being odd equal to 310=0.3.\frac 3{10} = 0.3. This means the probability it is even is 10.3=0.7.1-0.3=0.7.

Thus, the correct answer is D.

13.

At Megapolis Hospital one year, multiple-birth statistics were as follows: Sets of twins, triplets, and quadruplets accounted for 10001000 of the babies born. There were four times as many sets of triplets as sets of quadruplets, and there was three times as many sets of twins as sets of triplets. How many of these 10001000 babies were in sets of quadruplets?

 25 \ 25

 40 \ 40

 64 \ 64

 100 \ 100

 160 \ 160

Answer: D
Solution(s):

Let the number of sets of twins, triplets, and quadruplets be w,r,w,r, and qq respectively. We must find 4q4q which is 44 times the number of sets of children. Then, we know 2w+3r+4q=10002w+3r+4q=1000 as this is the number of total babies. Also, we have r=4qr = 4q and w=3rw=3r by the next statments. Thus, w=24q.w=24q. By substitution, we have 40q=1000,40q=1000, so 4q=100,4q=100, which is our answer.

Thus, the correct answer is D.

14.

How many squares whose sides are parallel to the axes and whose vertices have coordinates that are integers lie entirely within the region bounded by the line y=πx,y=\pi x, the line y=0.1y=-0.1 and the line x=5.1?x=5.1?

 30 \ 30

 41 \ 41

 45 \ 45

 50 \ 50

 57 \ 57

Answer: D
Solution(s):

The intersection of x=5.1x=5.1 and the other two lines have an xx-value of 5.1.5.1. The intersection of y=πxy=\pi x and y=0.1y = -0.1 has an xx-value of 0.1π,\dfrac{-0.1}\pi, which is greater than 1.-1.

Thus, the xx value of some point on the square must be between 00 and 55 inclusive. Within this bound, πx<0.1,\pi x < -0.1, so 0.1<y<πx.-0.1 < y < \pi x. This makes the largest points with the largest yy-value given an xx-value is (0,0),(0,0),(1,3),(1,3),(2,6),(2,6),(3,9),(3,9),(4,12),(4,12),(5,15).(5,15).

Now that we know the bounds of the triangle, we shall count the number of squares by looking at the top left corner of the square. We shall case on the size of the square.

If it is a 1×11 \times 1 square, then the yy value must be greater than or equal to 11 and the xx value must be less than or equal to 4.4. The number of possible values for this is 3+6+9+12=30.3+6+9+12=30.

If it is a 2×22 \times 2 square, then the yy value must be greater than or equal to 22 and the xx value must be less than or equal to 3.3. The number of possible values for this is 2+5+8=15.2+5+8=15.

If it is a 3×33 \times 3 square, then the yy value must be greater than or equal to 33 and the xx value must be less than or equal to 2.2. The number of possible values for this is 1+4=5.1+4=5.

The total is then 30+15+5=50.30+15+5=50.

Thus, the correct answer is D.

15.

All the numbers 1,1, 2,2, 3,3, 4,4, 5,5, 6,6, 7,7, 8,8, 99 are written in a 3×33\times3 array of squares, one number in each square, in such a way that if two numbers are consecutive then they occupy squares that share an edge. The numbers in the four corners add up to 18.18. What is the number in the center?

 5 \ 5

 6 \ 6

 7 \ 7

 8 \ 8

 9 \ 9

Answer: C
Solution(s):

We firstly claim that everything that is either in the center or a corner is odd. This is because every number is next to a consecutive number and therefore has the opposite parity as it.

Therefore, all of the points that are the center or a corner are the same parity. The are 55 such points, but only 44 even numbers, so their parity isn't even. Thus, they are all odd. This means the sum of the corners plus the center is 1+3+5+7+9=25.1+3+5+7+9=25.

Since the corners have a sum of 18,18, the center has a value of 2518=7.25-18=7.

Thus, the correct answer is C.

16.

The sum of an infinite geometric series is a positive number S,S, and the second term in the series is 1.1. What is the smallest possible value of S?S?

 1+52 \ \dfrac{1+\sqrt{5}}{2}

 2 \ 2

 5 \ \sqrt{5}

 3 \ 3

 4 \ 4

Answer: E
Solution(s):

Let the first value of the series be a,a, and let the ratio be r.r. Thus, S=a1r=arr(1r)=1r(1r).\begin{align*}S&=\dfrac{a}{1-r}\\ &= \dfrac{ar}{r(1-r)} \\&= \dfrac{1}{r(1-r)}.\end{align*} This means we have to find rr that maximizes r(1r)=0.25(r0.5)2.r(1-r)= 0.25-(r-0.5)^2. This maximization will happen with r=0.5.r=0.5.

Therefore, S=10.5(0.5)=4.S = \dfrac{1}{0.5(0.5)}= 4.

Thus, the correct answer is E.

17.

All the numbers 2,2,3, 3, 4,4, 5,5,6, 6,7 7 are assigned to the six faces of a cube, one number to each face. For each of the eight vertices of the cube, a product of three numbers is computed, where the three numbers are the numbers assigned to the three faces that include that vertex. What is the greatest possible value of the sum of these eight products?

 312 \ 312

 343 \ 343

 625 \ 625

 729 \ 729

 1680 \ 1680

Answer: D
Solution(s):

Suppose that the pairs of opposite sides are labelled (a1,a2),(a_1,a_2),(b1,b2),(b_1,b_2),(c1,c2).(c_1,c_2). For the numbers on the same side as a1,a_1, the total is a1(b1c1+b1c2+b2c2+b2c1)a_1(b_1c_1+b_1c_2 + b_2c_2+b_2c_1) =a1(b1+b2)(c1+c2).= a_1(b_1+b_2)(c_1+c_2). It is similar on the other side, so the total is: (a1+a2)(b1+b2)(c1+c2).(a_1+a_2)(b_1+b_2)(c_1+c_2). Then, 9=(a1+a2)(b1+b2)(c1+c2)39 = \dfrac{(a_1+a_2)(b_1+b_2)(c_1+c_2)}3 ((a1+a2)(b1+b2)(c1+c2))13\geq ((a_1+a_2)(b_1+b_2)(c_1+c_2))^{\frac 13} by AM-GM, so our value is at most 93=729.9^3=729. Also, if each number xx is opposite to 9x,9-x, we have the product as 729,729, which is a possible value. This makes it, by definition, the maximum.

Thus, the correct answer is D.

18.

In how many ways can 345345 be written as the sum of an increasing sequence of two or more consecutive positive integers?

 1 \ 1

 3 \ 3

 5 \ 5

 6 \ 6

 7 \ 7

Answer: B
Solution(s):

Let the smallest number be x.x. Then, let the size of the sequence be s.s. This makes the largest integer x+s1.x+s-1.

Thus, the average of every number is x+s12.x + \dfrac{s-1}2. This makes the sum of all the numbers s(x+s12)=345.s\left(x + \dfrac{s-1}2\right)=345. This is equivalent to s(2x+s1)=690,s(2x+s-1)=690, or equivalently: 2x+s1=690s2x+s-1=\dfrac{690}s

As xx and ss are integers, we know that in order for the left hand side of the equation to be an integer, ss must be a factor of 690.690.

Observing the 1616 factors of 690,690, observe that all factors strictly less than 3030 must work, as 34530=11.5,\dfrac{345}{30}=11.5, suggesting that 1111 and 1212 are the middle numbers in the sequence. However, a sequence of 3030 integers with the middle numbers of 1111 and 1212 must have negative numbers at the start of the sequence. However, this isn't allowed!

As such, all sequences of length 0<s<300\lt s\lt 30 work that are factors of 690.690. These are: 2,3,5,6,10,15,232,3,5,6,10,15,23

Thus, the correct answer is E.

19.

Rectangle ABCDABCD has AB=5AB=5 and BC=4.BC=4. Point EE lies on AB\overline{AB} so that EB=1,EB=1, point GG lies on BC\overline{BC} so that CG=1,CG=1, and point FF lies on CD\overline{CD} so that DF=2.DF=2. Segments AG\overline{AG} and AC\overline{AC} intersect EF\overline{EF} at QQ and P,P, respectively. What is the value of PQEF?\dfrac{PQ}{EF}?

 316 ~\dfrac{\sqrt{3}}{16}

 213 ~\dfrac{\sqrt{2}}{13}

 982 ~\dfrac{9}{82}

 1091 ~\dfrac{10}{91}

 19 ~\dfrac19

Answer: D
Solution(s):

Observe that the value AE=ABEBAE = AB-EB=51 = 5-1=4.=4. Also, the value FC=DCDFFC = DC-DF =52= 5-2=3.=3. Then, since AEFC,AE \mid \mid FC, we know AEPCFP\triangle AEP \sim \triangle CFP by angle angle symmetry. Thus, PFPE=FCEA=34.\dfrac{PF}{PE} = \dfrac{FC}{EA} = \dfrac 34 . This makes PFEF=33+4=37.\dfrac{PF}{EF} = \dfrac{3}{3+4} = \dfrac 37.

Then, let XX be the extension of AGAG to meet CD.CD. Since two sides are parallel, we have ADDX=BGAB.\dfrac{AD}{DX} = \dfrac{BG}{AB}. Thus, 4DX=35\dfrac{4}{DX} = \dfrac{3}{5} DX=203.DX = \dfrac{20}3. This makes FX=2032=143.FX = \dfrac{20}3 - 2 = \dfrac{14}3 . Then, since AEFX,AE \mid \mid FX, we know AEQXFQ\triangle AEQ \sim \triangle XFQ by angle angle symmetry. Therefore, QFQE=FXEA=1434=76.\dfrac{QF}{QE} = \dfrac{FX}{EA} = \dfrac {\frac{14}{3}}4 = \dfrac 76. This makes QFEF=77+6=713.\dfrac{QF}{EF} = \dfrac{7}{7+6} = \dfrac 7{13}.

As such, QPEF=QFEFPFEF\dfrac{QP}{EF} = \dfrac{QF}{EF} - \dfrac{PF}{EF} =71337= \dfrac{7}{13} - \frac 37 =1091.= \dfrac{10}{91}.

Thus, the correct answer is D.

20.

A dilation of the plane—that is, a size transformation with a positive scale factor—sends the circle of radius 22 centered at A(2,2)A(2,2) to the circle of radius 33 centered at A(5,6).A’(5,6). What distance does the origin O(0,0),O(0,0), move under this transformation?

 0 \ 0

 3 \ 3

 13 \ \sqrt{13}

 4 \ 4

 5 \ 5

Answer: C
Solution(s):

The line between the centers must be on the center of dialation as the center is dialated. Also, the dialation factor is 1.51.5 since the radius goes from 22 to 3.3. Let the center of dialation be C.C. Then, CA:CA=1:1.5.CA:CA' = 1:1.5. Thus, CA:AA=1:0.5=2:1.CA:AA'= 1:0.5= 2:1.

Therefore, we can go in the direction of the vector AAA'A twice from A.A. The vector means we go down 44 and left 3.3. Doing this twice from AA is (4,6).(-4,-6).

Then, CO:CO=1:1.5CO:CO' = 1:1.5 since it dialates, meaning CO:OO=1:0.5.CO:OO' = 1:0.5. This means the distance moved is the length of COCO divided by 2,2, which is equivalent to 522=13.\dfrac{\sqrt{52}}2 = \sqrt{13}.

Thus, the correct answer is C.

21.

What is the area of the region enclosed by the graph of the equation x2+y2=x+y?x^2+y^2=|x|+|y|?

 π+2 \ \pi+\sqrt{2}

 π+2 \ \pi+2

 π+22 \ \pi+2\sqrt{2}

 2π+2 \ 2\pi+\sqrt{2}

 2π+22 \ 2\pi+2\sqrt{2}

Answer: B
Solution(s):

x2+y2=x2+y2|x|^2+|y|^2 =x^2+y^2=x+y,=|x|+|y|, so (x0.5)2+(y0.5)2=0.5(|x|-0.5)^2+(|y|-0.5)^2 = 0.5 This means the graph is symmetric over all quandrants, so we could find the area of the region in just the first quadrant and multiply it by 4.4.

Thus, we could look at the area of (x0.5)2+(y0.5)2=0.5(x-0.5)^2+(y-0.5)^2 = 0.5 with x,y0.x,y \geq 0.

If x+y<1,x+y< 1, then we have a triangle with base and height of 1,1, making the area 0.5.0.5.

Since the line x+y=1x+y=1 goes through the center, the region x+y1x+y \geq 1 creates a semicircle of radius 0.5.\sqrt{0.5}. Thus, the area is πr22=π4.\dfrac{\pi r^2}2 = \dfrac{\pi} 4.

This means the total area in each quadrant is 0.5+π4.0.5 + \dfrac{\pi}{4}. Therefore, the total area is 4(0.5+π4)=2+π.4\left(0.5 + \dfrac{\pi}{4}\right) = 2 + \pi.

Thus, the correct answer is C.

22.

A set of teams held a round-robin tournament in which every team played every other team exactly once. Every team won 1010 games and lost 1010 games; there were no ties. How many sets of three teams {A,B,C}\{A, B, C\} were there in which AA beat B,B, BB beat C,C, and CC beat A?A?

 385 \ 385

 665 \ 665

 945 \ 945

 1140 \ 1140

 1330 \ 1330

Answer: A
Solution(s):

The total number of teams is 10+10+1=21.10+10+1=21. The total number of sets is therefore (213)=1330.\binom{21}{3} = 1330.

Now, we must subtract the total number of sets such that there is no cycle. This only happens if one team beats the other two teams. There are 2121 choices for the team that beat the other two and (102)=45\binom{10}{2} = 45 ways to choose the teams they beat. Thus, the total of non-cycles is 2145=945.21\cdot 45=945. This means the total number of cycles is 1330945=385.1330-945=385.

Thus, the correct answer is A.

23.

In regular hexagon ABCDEF,ABCDEF, points W,W, X,X, Y,Y, and ZZ are chosen on sides BC,\overline{BC}, CD,\overline{CD}, EF,\overline{EF}, and FA\overline{FA} respectively, so lines AB,AB, ZW,ZW, YX,YX, and EDED are parallel and equally spaced. What is the ratio of the area of hexagon WCXYFZWCXYFZ to the area of hexagon ABCDEF?ABCDEF?

 13 \ \dfrac{1}{3}

 1027 \ \dfrac{10}{27}

 1127 \ \dfrac{11}{27}

 49 \ \dfrac{4}{9}

 1327 \ \dfrac{13}{27}

Answer: C
Solution(s):

Consider the following diagram:

The shape is symmetric, so we can find the ratio of the areas of EFCDEFCD to the area of ZFCW.ZFCW. For this, we can extend FEFE and CDCD until they meet each other, and let this point be P.P.

Also, the distance from EDED to ZWZW is the same as the distance from ZWZW to YX,YX, and the distance from ZWZW to YXYX is twice the distance as the distance from ZWZW to FC.FC.

Thus, the distance from EDED to ZWZW is twice the distance as the distance from ZWZW to FC.FC. Let the altitude from PP to EDED be s.s. Then, if the altitude from PP to ZWZW is s+d,s+d, the altitude from PP to CFCF is s+1.5ds+1.5d because of the ratio. Then, since FPCPED,\triangle FPC \sim \triangle PED,FPEP=s+1.5ds. \dfrac{FP}{EP} = \dfrac{s+1.5d}{s}. Also, since FE=ED=PEFE= ED = PE as PEDPED is equilateral, we have 2s=s+1.5d,2s = s+1.5d, and so s=1.5d.s=1.5d.

Thus, the ratio between side lengths of PED\triangle PED and PZW\triangle PZW is 53\frac 53 which makes the ratio between the area 259.\frac{25}9. Also the ratio between side lengths of PED\triangle PED and PFC\triangle PFC is 2,2, which makes the ratio between the area 4.4.

This makes the area of EDCFEDCF 33 times the area of PED\triangle PED and the area of EDWZEDWZ 169\frac{16}{9} times the area of PED\triangle PED Therefore, the area of ZWCFZWCF is 119\frac{11}{9} times the area of PED.\triangle PED.

Thus, the ratio between ZWCFZWCF and EDCFEDCF is 1193=1127.\dfrac{\frac{11}9}{3} = \dfrac{11}{27} .

Thus, the correct answer is C.

24.

How many four-digit integers abcd,abcd, with a0,a \neq 0, have the property that the three two-digit integers ab<bc<cdab < bc < cd form an increasing arithmetic sequence?

One such number is 4692,4692, where a=4,a=4, b=6,b=6, c=9,c=9, and d=2.d=2.

 9 \ 9

 15 \ 15

 16 \ 16

 17 \ 17

 20 \ 20

Answer: D
Solution(s):

We know abca \leq b \leq c by analyzing ab<bc<cd.ab < bc < cd. Also, bcbc is the average of abab and cdcd so 10a+b+10c+d2=10b+c.\dfrac {10a+b+10c+d}2 = 10b+c. This means that 10(c2b+a)=b+2cd,10(c-2b+a)=-b+2c-d, making the right hand side a multiple of 10.10. Thus, it must be 00 or 1010 since it is digits that satisfy abc.a \leq b \leq c . Thus, we can case on that value.

Case 1: b+2cd=10-b+2c-d=102bac=12b-a-c=1

We can look at the possible values of c.c.

c=6:c=6: b+d=2,2b5=a.b+d=2, 2b-5=a. Thus, b2b\leq 2 from the first equation, but can't work for the second equation.

c=7:c=7: b+d=4,2b6=a.b+d=4, 2b-6=a. Thus, b4b\leq 4 from the first equation, and b>3b > 3 from the second eqution. This makes one case for b=4.b=4.

c=8:c=8: b+d=6,2b7=a.b+d=6, 2b-7=a. Thus, b6b\leq 6 from the first equation, and b>3b > 3 from the second eqution. This makes three cases for b=4,5,6.b=4,5,6.

c=9:c=9: b+d=8,2b8=a.b+d=8, 2b-8=a. Thus, b8b\leq 8 from the first equation, and b>4b > 4 from the second eqution. This makes four cases for b=5,6,7,8.b=5,6,7,8. This case has 88 solutions.

Case 2: b+2cd=0,-b+2c-d=0,2bac=0,2b-a-c=0, which means the digits are an arithmetic sequence.

If the difference is 1,1, then 1a61 \leq a \leq 6 makes 66 solutions.

If the difference is 2,2, then 1a31 \leq a \leq 3 makes 33 solutions. This case makes 99 solutions.

In total, the number of solutions is 8+9=17.8+9 = 17.

Thus, the correct answer is D.

25.

Let f(x)=k=210(kxkx),f(x)=\sum_{k=2}^{10}(\lfloor kx \rfloor -k \lfloor x \rfloor), where r\lfloor r \rfloor denotes the greatest integer less than or equal to r.r. How many distinct values does f(x)f(x) assume for x0?x \ge 0?

 32 \ 32

 36 \ 36

 45 \ 45

 46 \ 46

 infinitely many \ \text{infinitely many}

Answer: A
Solution(s):

First, note that x=x+{x}x = \lfloor x \rfloor + \{x\} where {x}\{x\} is the fractional part of x.x.

Thus, f(x)=k=210k(x+{x})kxf(x)=\sum_{k=2}^{10}\lfloor k(\lfloor x \rfloor + \{x\}) \rfloor -k \lfloor x \rfloor =k=210k{x}=\sum_{k=2}^{10}\lfloor k\{x\} \rfloor since kx\lfloor kx \rfloor is an integer. This means f(x)f(x) can only increase when the fractional part of kxkx becomes greater than ik\frac ik for some i<k.i < k.

Thus, we must count all possible fractions that are ik\frac ik for some kk less than or equal to 10.10. Note that we could exclusively count k6k \geq 6 as all k5k \leq 5 would have every fraction equal to a fraction where the denominator is greater than 6.6.

There are k1k-1 fractions for each k,k, so we have 9+8+7+6+5=359+8+7+6+5=35 fractions.

However, 48,\frac{4}{8},26, \frac 26,36, \frac 36 , 46\frac 46 are all duplicates from other greater k,k, so we subtract 4.4.

As such, we have 354=3135-4=31 possible values of f.f. Then, we add one more for f(x)=0.f(x)=0. This makes the total count equal to 31+1=32.31+1=32.

Thus, the correct answer is A.